General Question

finkelitis's avatar

A quadratic reciprocity question to finish my thesis?

Asked by finkelitis (1907points) April 10th, 2010

I’m turning in my thesis right now, and there’s one piece that should be fairly straightforward using quadratic reciprocity, but it seems so obvious I don’t want to bother (and not straightforward enough to actually do). So I thought I’d throw it out to fluther, and see if anyone wants to help me finish this thing out, because it feels like a bit of a chore for me, but I thought some folks here might enjoy it. I don’t know if it’s technically homework, but I thought you all might humor me here.

Here’s the problem: given some positive integer N, can you choose an integer M > N such that ±N is not a perfect square modulo M?

Clearly it can be done. I’d love to see a swift argument that says so.

Observing members: 0 Composing members: 0

5 Answers

lloydbird's avatar

Yes, it can be done. Yes….
Done.

gagara's avatar

ah, yes, I remember asking myself the same question, good old times…

BonusQuestion's avatar

Sorry, I just saw this question, but what if N is itself a perfect square?

BonusQuestion's avatar

It seems that if N is not a perfect square then the statement is true.

I am claiming that we can find M to be a prime number satisfying what you want. Assume for now that N is odd.

Using Legendre symbol we need to have (-1/M) = 1 & (N/M) = -1 since Legendre symbol is multiplicative.

To have (-1/M) =1 for a prime number M, it is enough to have M = 1 (mod 4).

Because Legendre symbol is multiplicative we may assume N is square free with at least one prime factor. Say N = p1 * p2…. pn.

I construct a prime number M such that (p1/M) = -1 and (pi/M) = 1 for any i >1.

By reciprocity law we have (pi/M) (M/pi) = 1, since M = 1 mod 4. So we should have (M/pi) = 1 for i>1 and (M/p1) = -1. For the first one we only need to take M = 1 mod pi.

To have the second equality just take M to be ANY non-square number (say a) mod p1. Now we have a series of equations mod different primes and 4. By Chinese Remainder Theorem there is a number b such that any M satisfying this system of equations equals b mod (4* p1 * p2…*pn)

M = 1 mod 4
M = 1 mod pi i>1
M = a mod p1

Now by Dirichlet’s Theorem this has a solution in prime numbers and we are done.

Now if N is square free but even with at least 2 prime factors, the argument remains the same. You just need to make sure p1 is not 2.

If N=2 then we need to have (2/M) = -1 and (-1/M)=1. But we know that (2/M) = -1 when M = 3 or 5 mod 8. So it is enough to take M to be a prime number with M = 5 mod 8.

finkelitis's avatar

@BonusQuestion to the rescue! This is precisely what I was looking for.

I’d like to encourage anyone who checks out this question to give some lurve to the answer above.

Answer this question

Login

or

Join

to answer.

This question is in the General Section. Responses must be helpful and on-topic.

Your answer will be saved while you login or join.

Have a question? Ask Fluther!

What do you know more about?
or
Knowledge Networking @ Fluther